Saltar al contenido principal
LibreTexts Español

4.10: Abarcando, Independencia Lineal y Base en R

  • Page ID
    114562
  • \( \newcommand{\vecs}[1]{\overset { \scriptstyle \rightharpoonup} {\mathbf{#1}} } \) \( \newcommand{\vecd}[1]{\overset{-\!-\!\rightharpoonup}{\vphantom{a}\smash {#1}}} \)\(\newcommand{\id}{\mathrm{id}}\) \( \newcommand{\Span}{\mathrm{span}}\) \( \newcommand{\kernel}{\mathrm{null}\,}\) \( \newcommand{\range}{\mathrm{range}\,}\) \( \newcommand{\RealPart}{\mathrm{Re}}\) \( \newcommand{\ImaginaryPart}{\mathrm{Im}}\) \( \newcommand{\Argument}{\mathrm{Arg}}\) \( \newcommand{\norm}[1]{\| #1 \|}\) \( \newcommand{\inner}[2]{\langle #1, #2 \rangle}\) \( \newcommand{\Span}{\mathrm{span}}\) \(\newcommand{\id}{\mathrm{id}}\) \( \newcommand{\Span}{\mathrm{span}}\) \( \newcommand{\kernel}{\mathrm{null}\,}\) \( \newcommand{\range}{\mathrm{range}\,}\) \( \newcommand{\RealPart}{\mathrm{Re}}\) \( \newcommand{\ImaginaryPart}{\mathrm{Im}}\) \( \newcommand{\Argument}{\mathrm{Arg}}\) \( \newcommand{\norm}[1]{\| #1 \|}\) \( \newcommand{\inner}[2]{\langle #1, #2 \rangle}\) \( \newcommand{\Span}{\mathrm{span}}\)\(\newcommand{\AA}{\unicode[.8,0]{x212B}}\)

    Resultados
    1. Determine el lapso de un conjunto de vectores y determine si un vector está contenido en un lapso especificado.
    2. Determinar si un conjunto de vectores es linealmente independiente.
    3. Comprender los conceptos de subespacio, base y dimensión.
    4. Encuentra el espacio de fila, el espacio de columna y el espacio nulo de una matriz.

    Al generar todas las combinaciones lineales de un conjunto de vectores se pueden obtener varios subconjuntos de los\(\mathbb{R}^{n}\) cuales llamamos subespacios. Por ejemplo, ¿qué conjunto de vectores en\(\mathbb{R}^{3}\) generar el\(XY\) -plano? ¿Cuál es el conjunto de vectores más pequeño que puedes encontrar? Las herramientas de expansión, independencia lineal y base son exactamente lo que se necesita para responder a estas y similares preguntas y son el foco de esta sección. La siguiente definición es esencial.

    Definición\(\PageIndex{1}\): Subset

    Dejar\(U\) y\(W\) ser conjuntos de vectores en\(\mathbb{R}^n\). Si todos los vectores en también\(U\) están adentro\(W\), decimos que\(U\) es un subconjunto de\(W\), denotado\[U \subseteq W\nonumber \]

    Conjunto de vectores de expansión

    Comenzamos esta sección con una definición.

    Definición\(\PageIndex{2}\): Span of a Set of Vectors

    La colección de todas las combinaciones lineales de un conjunto de vectores\(\{ \vec{u}_1, \cdots ,\vec{u}_k\}\) en\(\mathbb{R}^{n}\) se conoce como el lapso de estos vectores y se escribe como\(\mathrm{span} \{\vec{u}_1, \cdots , \vec{u}_k\}\).

    Considera el siguiente ejemplo.

    Ejemplo\(\PageIndex{1}\): Span of Vectors

    Describir el lapso de los vectores\(\vec{u}=\left[ \begin{array}{rrr} 1 & 1 & 0 \end{array} \right]^T\) y\(\vec{v}=\left[ \begin{array}{rrr} 3 & 2 & 0 \end{array} \right]^T \in \mathbb{R}^{3}\).

    Solución

    Se puede ver que cualquier combinación lineal de los vectores\(\vec{u}\) y\(\vec{v}\) produce un vector de la forma\(\left[ \begin{array}{rrr} x & y & 0 \end{array} \right]^T\) en el\(XY\) plano -.

    Además, cada vector en el\(XY\) plano es de hecho una combinación lineal de los vectores\(\vec{u}\) y\(\vec{v}\). Eso es porque\[\left[ \begin{array}{r} x \\ y \\ 0 \end{array} \right] = (-2x+3y) \left[ \begin{array}{r} 1 \\ 1 \\ 0 \end{array} \right] + (x-y)\left[ \begin{array}{r} 3 \\ 2 \\ 0 \end{array} \right]\nonumber \]

    Así\(\mathrm{span}\{\vec{u},\vec{v}\}\) es precisamente el\(XY\) -plano.

    Puedes convencerte de que ningún vector puede abarcar el\(XY\) plano -plano. De hecho, tómate un momento para considerar qué se entiende por el lapso de un solo vector.

    Sin embargo puedes hacer que el conjunto sea más grande si lo deseas. Por ejemplo, considere el conjunto más grande de vectores\(\{ \vec{u}, \vec{v}, \vec{w}\}\) donde\(\vec{w}=\left[ \begin{array}{rrr} 4 & 5 & 0 \end{array} \right]^T\). Dado que los dos primeros vectores ya abarcan todo el\(XY\) -plano, el span es una vez más precisamente el\(XY\) -plano y no se ha ganado nada. Por supuesto si agregas un nuevo vector como\(\vec{w}=\left[ \begin{array}{rrr} 0 & 0 & 1 \end{array} \right]^T\) entonces sí abarca un espacio diferente. ¿Cuál es el lapso de\(\vec{u}, \vec{v}, \vec{w}\) en este caso?

    La distinción entre los conjuntos\(\{ \vec{u}, \vec{v}\}\) y se\(\{ \vec{u}, \vec{v}, \vec{w}\}\) hará utilizando el concepto de independencia lineal.

    Considerar los vectores\(\vec{u}, \vec{v}\), y\(\vec{w}\) discutidos anteriormente. En el siguiente ejemplo, mostraremos cómo demostrar formalmente que\(\vec{w}\) está en el lapso de\(\vec{u}\) y\(\vec{v}\).

    Ejemplo\(\PageIndex{2}\): Vector in a Span

    Dejar\(\vec{u}=\left[ \begin{array}{rrr} 1 & 1 & 0 \end{array} \right]^T\) y\(\vec{v}=\left[ \begin{array}{rrr} 3 & 2 & 0 \end{array} \right]^T \in \mathbb{R}^{3}\). Espectáculo que\(\vec{w} = \left[ \begin{array}{rrr} 4 & 5 & 0 \end{array} \right]^{T}\) está en\(\mathrm{span} \left\{ \vec{u}, \vec{v} \right\}\).

    Solución

    Para que un vector esté en\(\mathrm{span} \left\{ \vec{u}, \vec{v} \right\}\), debe ser una combinación lineal de estos vectores. Si\(\vec{w} \in \mathrm{span} \left\{ \vec{u}, \vec{v} \right\}\), debemos ser capaces de encontrar escalares\(a,b\) tales que\[\vec{w} = a \vec{u} +b \vec{v}\nonumber \]

    Procedemos de la siguiente manera. \[\left[ \begin{array}{r} 4 \\ 5 \\ 0 \end{array} \right] = a \left[ \begin{array}{r} 1 \\ 1 \\ 0 \end{array} \right] + b \left[ \begin{array}{r} 3 \\ 2 \\ 0 \end{array} \right]\nonumber \]Esto equivale al siguiente sistema de ecuaciones\[\begin{aligned} a + 3b &= 4 \\ a + 2b &= 5\end{aligned}\]

    Resolvimos este sistema de la manera habitual, construyendo la matriz aumentada y la reducción de filas para encontrar la forma de fila-escalón reducida. \[\left[ \begin{array}{rr|r} 1 & 3 & 4 \\ 1 & 2 & 5 \end{array} \right] \rightarrow \cdots \rightarrow \left[ \begin{array}{rr|r} 1 & 0 & 7 \\ 0 & 1 & -1 \end{array} \right]\nonumber \]La solución es\(a=7, b=-1\). Esto quiere decir que\[\vec{w} = 7 \vec{u} - \vec{v}\nonumber \] por lo tanto podemos decir que\(\vec{w}\) está en\(\mathrm{span} \left\{ \vec{u}, \vec{v} \right\}\).

    Conjunto Linealmente Independiente de Vectores

    Ahora volvemos nuestra atención a la siguiente pregunta: ¿qué combinaciones lineales de un conjunto dado de vectores\(\{ \vec{u}_1, \cdots ,\vec{u}_k\}\) en\(\mathbb{R}^{n}\) arroja el vector cero? Claramente\(0\vec{u}_1 + 0\vec{u}_2+ \cdots + 0 \vec{u}_k = \vec{0}\), pero ¿es posible tener\(\sum_{i=1}^{k}a_{i}\vec{u}_{i}=\vec{0}\) sin que todos los coeficientes sean cero?

    Puedes crear ejemplos donde esto suceda fácilmente. Por ejemplo si\(\vec{u}_1=\vec{u}_2\), entonces\(1\vec{u}_1 - \vec{u}_2+ 0 \vec{u}_3 + \cdots + 0 \vec{u}_k = \vec{0}\), no importan los vectores\(\{ \vec{u}_3, \cdots ,\vec{u}_k\}\). 0Pero a veces puede ser más sutil.

    Ejemplo\(\PageIndex{3}\): Linearly Dependent Set of Vectors

    Considera los vectores\[\vec{u}_1=\left[ \begin{array}{rrr} 0 & 1 & -2 \end{array} \right]^T, \vec{u}_2=\left[ \begin{array}{rrr} 1 & 1 & 0 \end{array} \right]^T, \vec{u}_3=\left[ \begin{array}{rrr} -2 & 3 & 2 \end{array} \right]^T, \mbox{ and } \vec{u}_4=\left[ \begin{array}{rrr} 1 & -2 & 0 \end{array} \right]^T\nonumber \] en\(\mathbb{R}^{3}\).

    Luego verificar que\[1\vec{u}_1 +0 \vec{u}_2+ - \vec{u}_3 -2 \vec{u}_4 = \vec{0}\nonumber \]

    Se puede ver que la combinación lineal produce el vector cero pero tiene algunos coeficientes distintos de cero. Así definimos un conjunto de vectores para que sean linealmente dependientes si esto sucede.

    Definición\(\PageIndex{3}\): Linearly Dependent Set of Vectors

    Se dice que un conjunto de vectores distintos de cero\(\{ \vec{u}_1, \cdots ,\vec{u}_k\}\) en\(\mathbb{R}^{n}\) es linealmente dependiente si una combinación lineal de estos vectores sin que todos los coeficientes sean cero produce el vector cero.

    Tenga en cuenta que si\(\sum_{i=1}^{k}a_{i}\vec{u}_{i}=\vec{0}\) y algún coeficiente es distinto de cero\(a_1 \neq 0\), digamos, entonces\[\vec{u}_1 = \frac{-1}{a_1} \sum_{i=2}^{k}a_{i}\vec{u}_{i}\nonumber \] y así\(\vec{u}_1\) está en el lapso de los otros vectores. Y lo contrario claramente funciona también, así que obtenemos que un conjunto de vectores es linealmente dependiente precisamente cuando uno de sus vectores está en el lapso de los otros vectores de ese conjunto.

    En particular, puede mostrar que el vector\(\vec{u}_1\) en el ejemplo anterior está en el lapso de los vectores\(\{ \vec{u}_2, \vec{u}_3, \vec{u}_4 \}\).

    Si un conjunto de vectores NO es linealmente dependiente, entonces debe ser que cualquier combinación lineal de estos vectores que rinda el vector cero debe usar todos los coeficientes cero. Esta es una noción muy importante, y le damos su propio nombre de independencia lineal.

    Definición\(\PageIndex{4}\): Linearly Independent Set of Vectors

    Se dice que un conjunto de vectores distintos de cero\(\{ \vec{u}_1, \cdots ,\vec{u}_k\}\) en\(\mathbb{R}^{n}\) es linealmente independiente si cada vez\[\sum_{i=1}^{k}a_{i}\vec{u}_{i}=\vec{0}\nonumber \] que sigue que cada uno\(a_{i}=0\).

    Tenga en cuenta también que requerimos que todos los vectores sean distintos de cero para formar un conjunto linealmente independiente.

    Para ver esto en un entorno más familiar, forme la\(n \times k\) matriz\(A\) teniendo estos vectores como columnas. Entonces todo lo que estamos diciendo es que el conjunto\(\{ \vec{u}_1, \cdots ,\vec{u}_k\}\) es linealmente independiente precisamente cuando\(AX=0\) tiene sólo la solución trivial.

    Aquí hay un ejemplo.

    Ejemplo\(\PageIndex{4}\): Linearly Independent Vectors

    Considere los vectores\(\vec{u}=\left[ \begin{array}{rrr} 1 & 1 & 0 \end{array} \right]^T\),\(\vec{v}=\left[ \begin{array}{rrr} 1 & 0 & 1 \end{array} \right]^T\), y\(\vec{w}=\left[ \begin{array}{rrr} 0 & 1 & 1 \end{array} \right]^T\) en\(\mathbb{R}^{3}\). Verificar si el conjunto\(\{\vec{u}, \vec{v}, \vec{w}\}\) es linealmente independiente.

    Solución

    Entonces supongamos que tenemos unas combinaciones lineales\(a\vec{u} + b \vec{v} + c\vec{w} = \vec{0}\). Entonces se puede ver que esto sólo puede suceder con\(a=b=c=0\).

    Como se mencionó anteriormente, se puede formar equivalentemente la\(3 \times 3\) matriz\(A = \left[ \begin{array}{ccc} 1 & 1 & 0 \\ 1 & 0 & 1 \\ 0 & 1 & 1 \\ \end{array} \right]\), y demostrar que solo\(AX=0\) tiene la solución trivial.

    Por lo tanto, esto significa que el conjunto\(\left\{ \vec{u}, \vec{v}, \vec{w} \right\}\) es linealmente independiente.

    En términos de expansión, un conjunto de vectores es linealmente independiente si no contiene vectores innecesarios, eso no es vector está en el lapso de los demás.

    Así juntamos todo esto en el siguiente teorema importante.

    Teorema\(\PageIndex{1}\): Linear Independence as a Linear Combination

    Dejar\(\left\{\vec{u}_{1},\cdots ,\vec{u}_{k}\right\}\) ser una colección de vectores en\(\mathbb{R}^{n}\). Entonces los siguientes son equivalentes:

    1. Es linealmente independiente, es decir siempre que se\[\sum_{i=1}^{k}a_{i}\vec{u}_{i}=\vec{0}\nonumber \] deduce que cada coeficiente\(a_{i}=0\).
    2. Ningún vector está en el lapso de los demás.
    3. El sistema de ecuaciones lineales sólo\(AX=0\) tiene la solución trivial, donde\(A\) está la\(n \times k\) matriz teniendo estos vectores como columnas.

    La última frase de este teorema es útil ya que nos permite utilizar la forma fila-escalón reducida de una matriz para determinar si un conjunto de vectores es linealmente independiente. Que los vectores sean columnas de una matriz\(A\). Encuentra la forma de escalón de fila reducida de\(A\). Si cada columna tiene una inicial, entonces se deduce que los vectores son linealmente independientes.

    A veces nos referimos a la condición con respecto a las sumas de la siguiente manera: El conjunto de vectores,\(\left\{ \vec{u}_{1},\cdots ,\vec{u}_{k}\right\}\) es linealmente independiente si y solo si no hay una combinación lineal no trivial que sea igual al vector cero. Una combinación lineal no trivial es aquella en la que no todos los escalares son iguales a cero. Del mismo modo, una combinación lineal trivial es aquella en la que todos los escalares son iguales a cero.

    Aquí hay un ejemplo detallado en\(\mathbb{R}^{4}\).

    Ejemplo\(\PageIndex{5}\): Linear Independence

    Determinar si el conjunto de vectores dados por\[\left\{ \left[ \begin{array}{r} 1 \\ 2 \\ 3 \\ 0 \end{array} \right], \; \left[ \begin{array}{r} 2 \\ 1 \\ 0 \\ 1 \end{array} \right] , \; \left[ \begin{array}{r} 0 \\ 1 \\ 1 \\ 2 \end{array} \right] , \; \left[ \begin{array}{r} 3 \\ 2 \\ 2 \\ 0 \end{array} \right] \right\}\nonumber \] es linealmente independiente. Si es linealmente dependiente, expresa uno de los vectores como una combinación lineal de los otros.

    Solución

    En este caso la matriz del correspondiente sistema homogéneo de ecuaciones lineales es\[\left[ \begin{array}{rrrr|r} 1 & 2 & 0 & 3 & 0\\ 2 & 1 & 1 & 2 & 0 \\ 3 & 0 & 1 & 2 & 0 \\ 0 & 1 & 2 & 0 & 0 \end{array} \right]\nonumber \]

    La forma reducida de fila-escalón es\[\left[ \begin{array}{rrrr|r} 1 & 0 & 0 & 0 & 0 \\ 0 & 1 & 0 & 0 & 0 \\ 0 & 0 & 1 & 0 & 0 \\ 0 & 0 & 0 & 1 & 0 \end{array} \right]\nonumber \]

    y así cada columna es una columna pivote y el sistema correspondiente\(AX=0\) sólo tiene la solución trivial. Por lo tanto, estos vectores son linealmente independientes y no hay manera de obtener uno de los vectores como una combinación lineal de los otros.

    Considera otro ejemplo.

    Ejemplo\(\PageIndex{6}\): Linear Independence

    Determinar si el conjunto de vectores dados por\[\left\{ \left[ \begin{array}{r} 1 \\ 2 \\ 3 \\ 0 \end{array} \right], \; \left[ \begin{array}{r} 2 \\ 1 \\ 0 \\ 1 \end{array} \right], \; \left[ \begin{array}{r} 0 \\ 1 \\ 1 \\ 2 \end{array} \right], \; \left[ \begin{array}{r} 3 \\ 2 \\ 2 \\ -1 \end{array} \right] \right\}\nonumber \] es linealmente independiente. Si es linealmente dependiente, expresa uno de los vectores como una combinación lineal de los otros.

    Solución

    Formar la\(4 \times 4\) matriz\(A\) teniendo estos vectores como columnas:\[A= \left[ \begin{array}{rrrr} 1 & 2 & 0 & 3 \\ 2 & 1 & 1 & 2 \\ 3 & 0 & 1 & 2 \\ 0 & 1 & 2 & -1 \end{array} \right]\nonumber \] Entonces por Teorema\(\PageIndex{1}\), el conjunto dado de vectores es linealmente independiente exactamente si el sistema\(AX=0\) tiene solo la solución trivial.

    La matriz aumentada para este sistema y la correspondiente forma de fila-escalón reducida están dadas por\[\left[ \begin{array}{rrrr|r} 1 & 2 & 0 & 3 & 0 \\ 2 & 1 & 1 & 2 & 0 \\ 3 & 0 & 1 & 2 & 0 \\ 0 & 1 & 2 & -1 & 0 \end{array} \right] \rightarrow \cdots \rightarrow \left[ \begin{array}{rrrr|r} 1 & 0 & 0 & 1 & 0 \\ 0 & 1 & 0 & 1 & 0 \\ 0 & 0 & 1 & -1 & 0 \\ 0 & 0 & 0 & 0 & 0 \end{array} \right]\nonumber \] No todas las columnas de la matriz de coeficientes son columnas pivotantes y por lo tanto los vectores no son linealmente independientes. En este caso, decimos que los vectores son linealmente dependientes.

    De ello se deduce que hay infinitamente muchas soluciones a\(AX=0\), una de las cuales es\[\left[ \begin{array}{r} 1 \\ 1 \\ -1 \\ -1 \end{array} \right]\nonumber \] Por lo tanto podemos escribir\[1\left[ \begin{array}{r} 1 \\ 2 \\ 3 \\ 0 \end{array} \right] +1\left[ \begin{array}{r} 2 \\ 1 \\ 0 \\ 1 \end{array} \right] -1 \left[ \begin{array}{r} 0 \\ 1 \\ 1 \\ 2 \end{array} \right] -1 \left[ \begin{array}{r} 3 \\ 2 \\ 2 \\ -1 \end{array} \right] = \left[ \begin{array}{r} 0 \\ 0 \\ 0 \\ 0 \end{array} \right]\nonumber \]

    Esto se puede reorganizar de la siguiente manera\[1\left[ \begin{array}{r} 1 \\ 2 \\ 3 \\ 0 \end{array} \right] +1\left[ \begin{array}{r} 2 \\ 1 \\ 0 \\ 1 \end{array} \right] -1 \left[ \begin{array}{r} 0 \\ 1 \\ 1 \\ 2 \end{array} \right] =\left[ \begin{array}{r} 3 \\ 2 \\ 2 \\ -1 \end{array} \right]\nonumber \] Esto da el último vector como una combinación lineal de los tres primeros vectores.

    Observe que podríamos reorganizar esta ecuación para escribir cualquiera de los cuatro vectores como una combinación lineal de los otros tres.

    Cuando se le da un conjunto linealmente independiente de vectores, podemos determinar si los conjuntos relacionados son linealmente independientes.

    Ejemplo\(\PageIndex{7}\): Related Sets of Vectors

    Seamos\(\{ \vec{u},\vec{v},\vec{w}\}\) un conjunto independiente de\(\mathbb{R}^n\). ¿Es\(\{\vec{u}+\vec{v}, 2\vec{u}+\vec{w}, \vec{v}-5\vec{w}\}\) linealmente independiente?

    Solución

    Supongamos\(a(\vec{u}+\vec{v}) + b(2\vec{u}+\vec{w}) + c(\vec{v}-5\vec{w})=\vec{0}_n\) para algunos\(a,b,c\in\mathbb{R}\). Entonces\[(a+2b)\vec{u} + (a+c)\vec{v} + (b-5c)\vec{w}=\vec{0}_n.\nonumber \]

    Dado que\(\{\vec{u},\vec{v},\vec{w}\}\) es independiente,\[\begin{aligned} a + 2b & = 0 \\ a + c & = 0 \\ b - 5c & = 0 \end{aligned}\]

    Este sistema de tres ecuaciones en tres variables tiene la solución única\(a=b=c=0\). Por lo tanto,\(\{\vec{u}+\vec{v}, 2\vec{u}+\vec{w}, \vec{v}-5\vec{w}\}\) es independiente.

    El siguiente corolario se desprende del hecho de que si la matriz aumentada de un sistema homogéneo de ecuaciones lineales tiene más columnas que filas, el sistema tiene infinitamente muchas soluciones.

    Corolario\(\PageIndex{1}\): Linear Dependence in \(\mathbb{R}''\)

    Dejar\(\left\{ \vec{u}_{1},\cdots ,\vec{u}_{k}\right\}\) ser un conjunto de vectores en\(\mathbb{R}^{n}\). Si\(k>n\), entonces el conjunto es linealmente dependiente (es decir, NO linealmente independiente).

    Prueba

    Formar la\(n \times k\) matriz\(A\) teniendo los vectores\(\left\{ \vec{u}_{1},\cdots ,\vec{u}_{k}\right\}\) como sus columnas y supongamos\(k > n\). Entonces\(A\) tiene rango\(r \leq n <k\), por lo que el sistema\(AX=0\) tiene una solución no trivial y por lo tanto no linealmente independiente por Teorema\(\PageIndex{1}\).

    Ejemplo\(\PageIndex{8}\): Linear Dependence

    Considerar los vectores\[\left\{ \left[ \begin{array}{r} 1 \\ 4 \end{array} \right], \left[ \begin{array}{r} 2 \\ 3 \end{array} \right], \left[ \begin{array}{r} 3 \\ 2 \end{array} \right] \right\}\nonumber \] ¿Son estos vectores linealmente independientes?

    Solución

    Este conjunto contiene tres vectores en\(\mathbb{R}^2\). Por Corolario\(\PageIndex{1}\) estos vectores son linealmente dependientes. De hecho, podemos escribir\[(-1) \left[ \begin{array}{r} 1 \\ 4 \end{array} \right] + (2) \left[ \begin{array}{r} 2 \\ 3 \end{array} \right] = \left[ \begin{array}{r} 3 \\ 2 \end{array} \right]\nonumber \] mostrando que este conjunto es linealmente dependiente.

    El tercer vector en el ejemplo anterior está en el lapso de los dos primeros vectores. Podríamos encontrar una manera de escribir este vector como una combinación lineal de los otros dos vectores. Resulta que la combinación lineal que encontramos es la única, siempre que el conjunto sea linealmente independiente.

    Teorema\(\PageIndex{2}\): Unique Linear Combination

    \(U \subseteq\mathbb{R}^n\)Déjese ser un conjunto independiente. Entonces cualquier vector se\(\vec{x}\in\mathrm{span}(U)\) puede escribir únicamente como una combinación lineal de vectores de\(U\).

    Prueba

    Para probar este teorema, mostraremos que dos combinaciones lineales de vectores en\(U\) ese igual\(\vec{x}\) deben ser iguales. Vamos\(U =\{ \vec{u}_1, \vec{u}_2, \ldots, \vec{u}_k\}\). Supongamos que hay un vector\(\vec{x}\in \mathrm{span}(U)\) tal que\[\begin{aligned} \vec{x} & = s_1\vec{u}_1 + s_2\vec{u}_2 + \cdots + s_k\vec{u}_k, \mbox{ for some } s_1, s_2, \ldots, s_k\in\mathbb{R}, \mbox{ and} \\ \vec{x} & = t_1\vec{u}_1 + t_2\vec{u}_2 + \cdots + t_k\vec{u}_k, \mbox{ for some } t_1, t_2, \ldots, t_k\in\mathbb{R}.\end{aligned}\] Entonces\(\vec{0}_n=\vec{x}-\vec{x} = (s_1-t_1)\vec{u}_1 + (s_2-t_2)\vec{u}_2 + \cdots + (s_k-t_k)\vec{u}_k\).

    Ya que\(U\) es independiente, la única combinación lineal que se desvanece es la trivial, así que\(s_i-t_i=0\) para todos\(i\),\(1\leq i\leq k\).

    Por lo tanto,\(s_i=t_i\) para todos\(i\)\(1\leq i\leq k\),, y la representación es única.Dejemos\(U \subseteq\mathbb{R}^n\) ser un conjunto independiente. Entonces cualquier vector se\(\vec{x}\in\mathrm{span}(U)\) puede escribir únicamente como una combinación lineal de vectores de\(U\).

    Supongamos que\(\vec{u},\vec{v}\) y\(\vec{w}\) son vectores distintos de cero en\(\mathbb{R}^3\), y eso\(\{ \vec{v},\vec{w}\}\) es independiente. Considera el conjunto\(\{ \vec{u},\vec{v},\vec{w}\}\). ¿Cuándo podemos saber que este conjunto es independiente? Resulta que esto sigue exactamente cuándo\(\vec{u}\not\in\mathrm{span}\{\vec{v},\vec{w}\}\).

    Ejemplo\(\PageIndex{9}\)

    Supongamos que\(\vec{u},\vec{v}\) y\(\vec{w}\) son vectores distintos de cero en\(\mathbb{R}^3\), y eso\(\{ \vec{v},\vec{w}\}\) es independiente. Demostrar que\(\{ \vec{u},\vec{v},\vec{w}\}\) es independiente si y solo si\(\vec{u}\not\in\mathrm{span}\{\vec{v},\vec{w}\}\).

    Solución

    Si\(\vec{u}\in\mathrm{span}\{\vec{v},\vec{w}\}\), entonces existen\(a,b\in\mathbb{R}\) para que\(\vec{u}=a\vec{v} + b\vec{w}\). Esto implica que\(\vec{u}-a\vec{v} - b\vec{w}=\vec{0}_3\), así\(\vec{u}-a\vec{v} - b\vec{w}\) es una combinación lineal no trivial de\(\{ \vec{u},\vec{v},\vec{w}\}\) que se desvanece, y por lo tanto\(\{ \vec{u},\vec{v},\vec{w}\}\) es dependiente.

    Ahora supongamos eso\(\vec{u}\not\in\mathrm{span}\{\vec{v},\vec{w}\}\), y supongamos que existe\(a,b,c\in\mathbb{R}\) tal que\(a\vec{u}+b\vec{v}+c\vec{w}=\vec{0}_3\). Si\(a\neq 0\), entonces\(\vec{u}=-\frac{b}{a}\vec{v}-\frac{c}{a}\vec{w}\), y\(\vec{u}\in\mathrm{span}\{\vec{v},\vec{w}\}\), una contradicción. Por lo tanto\(a=0\),, implicando eso\(b\vec{v}+c\vec{w}=\vec{0}_3\). Dado que\(\{ \vec{v},\vec{w}\}\) es independiente\(b=c=0\), y así\(a=b=c=0\), es decir, la única combinación lineal de\(\vec{u},\vec{v}\) y\(\vec{w}\) que se desvanece es la trivial.

    Por lo tanto,\(\{ \vec{u},\vec{v},\vec{w}\}\) es independiente.

    Considera el siguiente teorema útil.

    Teorema\(\PageIndex{3}\): Invertible Matrices

    Dejar\(A\) ser una\(n \times n\) matriz invertible. Entonces las columnas de\(A\) son independientes y abarcan\(\mathbb{R}^n\). Del mismo modo, las filas de\(A\) son independientes y abarcan el conjunto de todos los\(1 \times n\) vectores.

    Este teorema también nos permite determinar si una matriz es invertible. Si una\(n \times n\) matriz\(A\) tiene columnas que son independientes, o abarcan\(\mathbb{R}^n\), entonces se deduce que\(A\) es invertible. Si tiene filas que son independientes, o abarcan el conjunto de todos los\(1 \times n\) vectores, entonces\(A\) es invertible.

    Una breve aplicación a la química

    En la siguiente sección se aplican los conceptos de expansión e independencia lineal al tema de la química.

    Cuando se trabaja con reacciones químicas, a veces hay un gran número de reacciones y algunas son en cierto sentido redundantes. Supongamos que tiene las siguientes reacciones químicas. \[\begin{array}{c} CO+\frac{1}{2}O_{2}\rightarrow CO_{2} \\ H_{2}+\frac{1}{2}O_{2}\rightarrow H_{2}O \\ CH_{4}+\frac{3}{2}O_{2}\rightarrow CO+2H_{2}O \\ CH_{4}+2O_{2}\rightarrow CO_{2}+2H_{2}O \end{array}\nonumber \]Aquí hay cuatro reacciones químicas pero no son reacciones independientes. Hay cierta redundancia. ¿Cuáles son las reacciones independientes? ¿Hay alguna manera de considerar una lista más corta de reacciones? Para analizar esta situación, podemos escribir las reacciones en una matriz de la siguiente manera\[\left[ \begin{array}{cccccc} CO & O_{2} & CO_{2} & H_{2} & H_{2}O & CH_{4} \\ 1 & 1/2 & -1 & 0 & 0 & 0 \\ 0 & 1/2 & 0 & 1 & -1 & 0 \\ -1 & 3/2 & 0 & 0 & -2 & 1 \\ 0 & 2 & -1 & 0 & -2 & 1 \end{array} \right]\nonumber \]

    Cada fila contiene los coeficientes de los elementos respectivos en cada reacción. Por ejemplo, la fila superior de números proviene de la\(CO+\frac{1}{2}O_{2}-CO_{2}=0\) cual representa la primera de las reacciones químicas.

    Podemos escribir estos coeficientes en la siguiente matriz En\[\left[ \begin{array}{rrrrrr} 1 & 1/2 & -1 & 0 & 0 & 0 \\ 0 & 1/2 & 0 & 1 & -1 & 0 \\ -1 & 3/2 & 0 & 0 & -2 & 1 \\ 0 & 2 & -1 & 0 & -2 & 1 \end{array} \right]\nonumber \] lugar de enumerar todas las reacciones como se indicó anteriormente, sería más eficiente enumerar solo aquellas que son independientes desechando lo que es redundante. Podemos utilizar los conceptos de la sección anterior para lograrlo.

    Primero, tome la forma reducida de fila-escalón de la matriz anterior. \[\left[ \begin{array}{rrrrrr} 1 & 0 & 0 & 3 & -1 & -1 \\ 0 & 1 & 0 & 2 & -2 & 0 \\ 0 & 0 & 1 & 4 & -2 & -1 \\ 0 & 0 & 0 & 0 & 0 & 0 \end{array} \right]\nonumber \]Las tres filas superiores representan reacciones “independientes” que provienen de las cuatro reacciones originales. Se puede obtener cada una de las cuatro filas originales de la matriz dada anteriormente tomando una combinación lineal adecuada de filas de esta matriz de escalón reducido.

    Con la reacción redundante eliminada, podemos considerar las reacciones simplificadas como las siguientes ecuaciones\[\begin{array}{c} CO+3H_{2}-1H_{2}O-1CH_{4}=0 \\ O_{2}+2H_{2}-2H_{2}O=0 \\ CO_{2}+4H_{2}-2H_{2}O-1CH_{4}=0 \end{array}\nonumber \] En términos de la notación original, estas son las reacciones\[\begin{array}{c} CO+3H_{2}\rightarrow H_{2}O+CH_{4} \\ O_{2}+2H_{2}\rightarrow 2H_{2}O \\ CO_{2}+4H_{2}\rightarrow 2H_{2}O+CH_{4} \end{array}\nonumber \]

    Estas tres reacciones proporcionan un sistema equivalente a las cuatro ecuaciones originales. La idea es que, en términos de lo que sucede químicamente, obtengas la misma información con la lista más corta de reacciones. Tal simplificación es especialmente útil cuando se trata de listas muy grandes de reacciones que pueden resultar de pruebas experimentales.

    Subespacios y Bases

    El objetivo de esta sección es desarrollar una comprensión de un subespacio de\(\mathbb{R}^n\). Antes de considerar una definición precisa, primero examinamos la prueba subespacial que se da a continuación.

    Teorema\(\PageIndex{4}\): Subspace Test

    Un subconjunto\(V\) de\(\mathbb{R}^n\) es un subespacio de\(\mathbb{R}^n\) si

    1. el vector cero de\(\mathbb{R}^n\),\(\vec{0}_n\), está en\(V\);
    2. \(V\)se cierra bajo adición, es decir, para todos\(\vec{u},\vec{w}\in V\),\(\vec{u}+\vec{w}\in V\);
    3. \(V\)se cierra bajo multiplicación escalar, es decir, para todos\(\vec{u}\in V\) y\(k\in\mathbb{R}\),\(k\vec{u}\in V\).

    Esta prueba nos permite determinar si un conjunto dado es un subespacio de\(\mathbb{R}^n\). Observe que el subconjunto\(V = \left\{ \vec{0} \right\}\) es un subespacio de\(\mathbb{R}^n\) (llamado el subespacio cero), como lo es\(\mathbb{R}^n\) en sí mismo. Un subespacio que no es el subespacio cero de\(\mathbb{R}^n\) se conoce como un subespacio apropiado.

    Un subespacio es simplemente un conjunto de vectores con la propiedad de que las combinaciones lineales de estos vectores permanecen en el conjunto. Geométricamente en\(\mathbb{R}^{3}\), resulta que un subespacio puede ser representado por el origen como un solo punto, líneas y planos que contienen el origen, o bien por todo el espacio\(\mathbb{R}^{3}\).

    Considera el siguiente ejemplo de una línea en\(\mathbb{R}^3\).

    Ejemplo\(\PageIndex{10}\): Subspace of \(\mathbb{R}^3\)

    En\(\mathbb{R}^3\), la línea\(L\) a través del origen que es paralela al vector\({\vec{d}}= \left[ \begin{array}{r} -5 \\ 1 \\ -4 \end{array}\right]\) tiene ecuación (vector)\(\left[ \begin{array}{r} x \\ y \\ z \end{array}\right] =t\left[ \begin{array}{r} -5 \\ 1 \\ -4 \end{array}\right], t\in\mathbb{R}\), entonces\[L=\left\{ t{\vec{d}} ~|~ t\in\mathbb{R}\right\}.\nonumber \] entonces\(L\) es un subespacio de\(\mathbb{R}^3\).

    Solución

    Utilizando la prueba subespacial dada anteriormente podemos verificar que\(L\) es un subespacio de\(\mathbb{R}^3\).

    • Primero:\(\vec{0}_3\in L\) desde\(0\vec{d}=\vec{0}_3\).
    • Supongamos\(\vec{u},\vec{v}\in L\). Entonces por definición,\(\vec{u}=s\vec{d}\) y\(\vec{v}=t\vec{d}\), para algunos\(s,t\in\mathbb{R}\). Así\[\vec{u}+\vec{v} = s\vec{d}+t\vec{d} = (s+t)\vec{d}.\nonumber \] Desde\(s+t\in\mathbb{R}\),\(\vec{u}+\vec{v}\in L\); es decir,\(L\) se cierra bajo adición.
    • Supongamos\(\vec{u}\in L\) y\(k\in\mathbb{R}\) (\(k\)es un escalar). Entonces\(\vec{u}=t\vec{d}\), para algunos\(t\in\mathbb{R}\), entonces\[k\vec{u}=k(t\vec{d})=(kt)\vec{d}.\nonumber \]\(kt\in\mathbb{R}\) Since,\(k\vec{u}\in L\); es decir,\(L\) se cierra bajo multiplicación escalar.

    Dado que\(L\) satisface todas las condiciones de la prueba subespacial, se deduce que\(L\) es un subespacio.

    Tenga en cuenta que no hay nada especial en el vector\(\vec{d}\) utilizado en este ejemplo; la misma prueba funciona para cualquier vector distinto de cero\(\vec{d}\in\mathbb{R}^3\), por lo que cualquier línea a través del origen es un subespacio de\(\mathbb{R}^3\).

    Ahora estamos preparados para examinar la definición precisa de un subespacio de la siguiente manera.

    Definición\(\PageIndex{5}\): Subspace

    Dejar\(V\) ser una colección no vacía de vectores en\(\mathbb{R}^{n}.\) Entonces\(V\) se llama un subespacio si siempre\(a\) y\(b\) son escalares y\(\vec{u}\) y\(\vec{v}\) son vectores en\(V,\) la combinación lineal también\(a \vec{u}+ b \vec{v}\) está en\(V\).

    De manera más general, esto significa que un subespacio contiene la extensión de cualquier vector de colección finita en ese subespacio. Resulta que en\(\mathbb{R}^{n}\), un subespacio es exactamente el lapso de finitamente muchos de sus vectores.

    Teorema\(\PageIndex{5}\): Subspaces are Spans

    Dejar\(V\) ser una colección no vacía de vectores en\(\mathbb{R}^{n}.\) Entonces\(V\) es un subespacio de\(\mathbb{R}^{n}\) si y solo si existen vectores\(\left\{ \vec{u}_{1},\cdots ,\vec{u}_{k}\right\}\) en\(V\) tal que\[V= \mathrm{span}\left\{ \vec{u}_{1},\cdots ,\vec{u}_{k}\right\}\nonumber \] además, dejemos\(W\) ser otro subespacio de\(\mathbb{R}^n\) y supongamos\(\left\{ \vec{u}_{1},\cdots ,\vec{u}_{k}\right\} \in W\). Entonces se deduce que\(V\) es un subconjunto de\(W\).

    Obsérvese que dado que\(W\) es arbitrario, la afirmación que\(V \subseteq W\) significa que cualquier otro subespacio del\(\mathbb{R}^n\) que contenga estos vectores también contendrá\(V\).

    Prueba

    Primero mostramos que si\(V\) es un subespacio, entonces se puede escribir como\(V= \mathrm{span}\left\{ \vec{u}_{1},\cdots ,\vec{u}_{k}\right\}\). Elija un vector\(\vec{u}_{1}\) en\(V\). Si\(V= \mathrm{span}\left\{ \vec{u}_{1}\right\} ,\) entonces has encontrado tu lista de vectores y estás listo. Si\(V\neq \mathrm{span}\left\{ \vec{u}_{1}\right\} ,\) entonces existe\(\vec{u}_{2}\) un vector del\(V\) cual no está en\(\mathrm{span}\left\{ \vec{u}_{1}\right\} .\) Considerar\(\mathrm{span}\left\{ \vec{u}_{1},\vec{u}_{2}\right\}.\) Si\(V=\mathrm{span}\left\{ \vec{u}_{1},\vec{u}_{2}\right\}\), ya terminamos. De lo contrario, escoja\(\vec{u}_{3}\) no en\(\mathrm{span}\left\{ \vec{u}_{1},\vec{u}_{2}\right\} .\) Continuar de esta manera. Tenga en cuenta que dado que\(V\) es un subespacio, cada uno de estos vanos está contenido en\(V\). El proceso debe detenerse con\(\vec{u}_{k}\) para algunos\(k\leq n\) por Corolario\(\PageIndex{1}\), y así\(V=\mathrm{span}\left\{ \vec{u}_{1},\cdots , \vec{u}_{k}\right\}\).

    Ahora supongamos\(V=\mathrm{span}\left\{ \vec{u}_{1},\cdots , \vec{u}_{k}\right\}\), debemos demostrar que esto es un subespacio. Así que vamos\(\sum_{i=1}^{k}c_{i}\vec{u}_{i}\)\(\sum_{i=1}^{k}d_{i}\vec{u}_{i}\) a ser dos vectores adentro\(V\), y let\(a\) y\(b\) ser dos escalares. Entonces\[a \sum_{i=1}^{k}c_{i}\vec{u}_{i}+ b \sum_{i=1}^{k}d_{i}\vec{u}_{i}= \sum_{i=1}^{k}\left( a c_{i}+b d_{i}\right) \vec{u}_{i}\nonumber \] que es uno de los vectores en\(\mathrm{span}\left\{ \vec{u}_{1},\cdots , \vec{u}_{k}\right\}\) y por lo tanto está contenido en\(V\). Esto demuestra que\(\mathrm{span}\left\{ \vec{u}_{1},\cdots ,\vec{u}_{k}\right\}\) tiene las propiedades de un subespacio.

    Para probarlo\(V \subseteq W\), lo demostramos si\(\vec{u}_i\in V\), entonces\(\vec{u}_i \in W\).

    Supongamos\(\vec{u}\in V\). Entonces\(\vec{u}=a_1\vec{u}_1 + a_2\vec{u}_2 + \cdots + a_k\vec{u}_k\) para algunos\(a_i\in\mathbb{R}\),\(1\leq i\leq k\). Dado que\(W\) contienen cada uno\(\vec{u}_i\) y\(W\) es un espacio vectorial, de ello se deduce que\(a_1\vec{u}_1 + a_2\vec{u}_2 + \cdots + a_k\vec{u}_k \in W\).

    Dado que los vectores que\(\vec{u}_i\) construimos en la prueba anterior no están en el lapso de los vectores anteriores (por definición), deben ser linealmente independientes y así obtenemos el siguiente corolario.

    Corolario\(\PageIndex{2}\): Subspaces are Spans of Independent Vectors

    Si\(V\) es un subespacio de\(\mathbb{R}^{n},\) entonces existen vectores linealmente independientes\(\left\{ \vec{u}_{1},\cdots ,\vec{u}_{k}\right\}\) en\(V\) tal que\(V=\mathrm{span}\left\{ \vec{u}_{1},\cdots ,\vec{u}_{k}\right\}\).

    En resumen, los subespacios de\(\mathbb{R}^{n}\) consisten en tramos de finitos, colecciones linealmente independientes de vectores de\(\mathbb{R}^{n}\). Tal colección de vectores se llama base.

    Definición\(\PageIndex{6}\): Basis of a Subspace

    Que\(V\) sea un subespacio de\(\mathbb{R}^{n}\). Entonces\(\left\{ \vec{u}_{1},\cdots ,\vec{u}_{k}\right\}\) es una base para\(V\) si se mantienen las siguientes dos condiciones.

    1. \(\mathrm{span}\left\{ \vec{u}_{1},\cdots ,\vec{u}_{k}\right\} =V\)
    2. \(\left\{ \vec{u}_{1},\cdots ,\vec{u}_{k}\right\}\)es linealmente independiente

    Tenga en cuenta que el plural de base es bases.

    El siguiente es un ejemplo sencillo pero muy útil de una base, llamada base estándar.

    Definición\(\PageIndex{7}\): Standard Basis of \(\mathbb{R}^n\)

    Dejar\(\vec{e}_i\) ser el vector en el\(\mathbb{R}^n\) que tiene a\(1\) en la\(i^{th}\) entrada y ceros en otra parte, esa es la\(i^{th}\) columna de la matriz de identidad. Entonces la colección\(\left\{\vec{e}_1, \vec{e}_2, \cdots, \vec{e}_n \right\}\) es una base para\(\mathbb{R}^n\) y se llama la base estándar de\(\mathbb{R}^n\).

    El teorema principal sobre las bases no es sólo que existen, sino que deben ser del mismo tamaño. Para mostrar esto, necesitaremos el siguiente resultado fundamental, llamado Teorema de Intercambio.

    Teorema\(\PageIndex{6}\): Exchange Theorem

    Supongamos que\(\left\{ \vec{u}_{1},\cdots ,\vec{u}_{r}\right\}\) es un conjunto linealmente independiente de vectores en\(\mathbb{R}^n\), y cada uno\(\vec{u}_{k}\) está contenido\(s\geq r.\)
    en\(\mathrm{span}\left\{ \vec{v}_{1},\cdots ,\vec{v}_{s}\right\}\) Entonces En palabras, los conjuntos de expansión tienen al menos tantos vectores como conjuntos linealmente independientes.

    Prueba

    Ya que cada uno\(\vec{u}_j\) está adentro\(\mathrm{span}\left\{ \vec{v}_{1},\cdots ,\vec{v}_{s}\right\}\), existen escalares\(a_{ij}\) tales que\[\vec{u}_{j}=\sum_{i=1}^{s}a_{ij}\vec{v}_{i}\nonumber \] Supongamos para una contradicción que\(s<r\). Entonces la matriz\(A = \left[ a_{ij} \right]\) tiene menos filas,\(s\) que columnas,\(r\). Entonces el sistema\(AX=0\) tiene una solución no trivial\(\vec{d}\), es decir hay\(\vec{d}\neq \vec{0}\) tal que\(A\vec{d}=\vec{0}\). Es decir,\[\sum_{j=1}^{r}a_{ij}d_{j}=0,\;i=1,2,\cdots ,s\nonumber \] Por lo tanto, lo\[\begin{aligned} \sum_{j=1}^{r}d_{j}\vec{u}_{j} &=\sum_{j=1}^{r}d_{j}\sum_{i=1}^{s}a_{ij} \vec{v}_{i} \\ &=\sum_{i=1}^{s}\left( \sum_{j=1}^{r}a_{ij}d_{j}\right) \vec{v} _{i}=\sum_{i=1}^{s}0\vec{v}_{i}=0\end{aligned}\] que contradice el supuesto de que\(\left\{ \vec{u}_{1},\cdots ,\vec{u}_{r}\right\}\) es linealmente independiente, porque no todos los\(d_{j}\) son cero. Así esa contradicción lo indica\(s\geq r\).

    Ahora estamos listos para demostrar que dos bases cualesquiera son del mismo tamaño.

    Teorema\(\PageIndex{7}\): Bases of \(\mathbb{R}^{n}\) are of the Same Size

    Dejar\(V\) ser un subespacio de\(\mathbb{R}^{n}\) con dos bases\(B_1\) y\(B_2\). Supongamos que\(B_1\) contiene\(s\) vectores y\(B_2\) contiene\(r\) vectores. Entonces\(s=r.\)

    Prueba

    Esto se desprende enseguida del Teorema 9.4.4. En efecto, observe que\(B_1 = \left\{ \vec{u}_{1},\cdots ,\vec{u}_{s}\right\}\) es un conjunto de expansión para\(V\) mientras que\(B_2 = \left\{ \vec{v}_{1},\cdots ,\vec{v}_{r}\right\}\) es linealmente independiente, por lo que\(s \geq r.\) Similarmente\(B_2 = \left\{ \vec{v}_{1},\cdots ,\vec{v} _{r}\right\}\) es un conjunto de expansión para\(V\) mientras\(B_1 = \left\{ \vec{u}_{1},\cdots , \vec{u}_{s}\right\}\) es linealmente independiente, entonces\(r\geq s\).

    Ahora se puede afirmar la siguiente definición.

    Definición\(\PageIndex{8}\): Dimension of a Subspace

    Que\(V\) sea un subespacio de\(\mathbb{R}^{n}\). Entonces la dimensión de\(V\), escrito\(\mathrm{dim}(V)\) se define como el número de vectores en una base.

    Sigue el siguiente resultado.

    Corolario\(\PageIndex{3}\): Dimension of \(\mathbb{R}^n\)

    La dimensión de\(\mathbb{R}^{n}\) es\(n.\)

    Prueba

    Solo necesitas exhibir una base para la\(\mathbb{R}^{n}\) cual tenga\(n\) vectores. Tal base es la base estándar\(\left\{ \vec{e}_{1},\cdots , \vec{e}_{n}\right\}\).

    Considera el siguiente ejemplo.

    Ejemplo\(\PageIndex{11}\): Basis of Subspace

    Vamos\[V=\left\{ \left[\begin{array}{c} a\\ b\\ c\\ d\end{array}\right]\in\mathbb{R}^4 ~:~ a-b=d-c \right\}.\nonumber \] Mostrar que\(V\) es un subespacio de\(\mathbb{R}^4\), encontrar una base de\(V\), y encontrar\(\dim(V)\).

    Solución

    La condición\(a-b=d-c\) es equivalente a la condición\(a=b-c+d\), por lo que podemos escribir

    \[V =\left\{ \left[\begin{array}{c} b-c+d\\ b\\ c\\ d\end{array}\right] ~:~b,c,d \in\mathbb{R} \right\} = \left\{ b\left[\begin{array}{c} 1\\ 1\\ 0\\ 0\end{array}\right] +c\left[\begin{array}{c} -1\\ 0\\ 1\\ 0\end{array}\right] +d\left[\begin{array}{c} 1\\ 0\\ 0\\ 1\end{array}\right] ~:~ b,c,d\in\mathbb{R} \right\}\nonumber \]

    Esto demuestra que\(V\) es un subespacio de\(\mathbb{R}^4\), desde\(V=\mathrm{span}\{ \vec{u}_1, \vec{u}_2, \vec{u}_3 \}\) donde

    \[\vec{u}_1 = \left[\begin{array}{r} 1 \\ 1 \\ 0 \\ 0 \end{array}\right], \vec{u}_2 = \left[\begin{array}{r} -1 \\ 0 \\ 1 \\ 0 \end{array}\right], \vec{u}_3 = \left[\begin{array}{r} 1 \\ 0 \\ 0 \\ 1 \end{array}\right]\nonumber \]

    Además,

    \[\left\{ \left[\begin{array}{c} 1\\ 1\\ 0\\ 0\end{array}\right], \left[\begin{array}{c} -1\\ 0\\ 1\\ 0\end{array}\right], \left[\begin{array}{c} 1\\ 0\\ 0\\ 1\end{array}\right] \right\}\nonumber \]es linealmente independiente, como puede verse tomando la forma de fila-escalón reducido de la matriz cuyas columnas son\(\vec{u}_1, \vec{u}_2\) y\(\vec{u}_3\).

    \[\left[\begin{array}{rrr} 1 & -1 & 1 \\ 1 & 0 & 0 \\ 0 & 1 & 0 \\ 0 & 0 & 1 \end{array}\right] \rightarrow \left[\begin{array}{rrr} 1 & 0 & 0 \\ 0 & 1 & 0 \\ 0 & 0 & 1 \\ 0 & 0 & 0 \end{array}\right]\nonumber \]

    Dado que cada columna de la matriz de forma fila-escalón reducida tiene una inicial, las columnas son linealmente independientes.

    Por lo tanto\(\{ \vec{u}_1, \vec{u}_2, \vec{u}_3 \}\) es linealmente independiente y abarca\(V\), por lo que es una base de\(V\). De ahí\(V\) que tenga la dimensión tres.

    Continuamos declarando más propiedades de un conjunto de vectores en\(\mathbb{R}^{n}\).

    Corolario\(\PageIndex{4}\): Linearly Independent and Spanning Sets in \(\mathbb{R}^{n}\)

    Las siguientes propiedades se mantienen en\(\mathbb{R}^{n}\):

    • Supongamos que\(\left\{ \vec{u}_{1},\cdots ,\vec{u}_{n}\right\}\) es linealmente independiente. Entonces\(\left\{ \vec{u}_{1},\cdots ,\vec{u}_{n}\right\}\) es una base para\(\mathbb{R}^{n}\).
    • Supongamos que\(\left\{ \vec{u}_{1},\cdots ,\vec{u}_{m}\right\}\) abarca\(\mathbb{R}^{n}.\) Entonces\(m\geq n.\)
    • Si\(\left\{ \vec{u}_{1},\cdots ,\vec{u}_{n}\right\}\) abarca\(\mathbb{R}^{n},\) entonces\(\left\{ \vec{u}_{1},\cdots ,\vec{u}_{n}\right\}\) es linealmente independiente.
    Prueba

    Supongamos primero que\(\left\{ \vec{u}_{1},\cdots ,\vec{u}_{n}\right\}\) es linealmente independiente, y necesitamos demostrar que este conjunto abarca\(\mathbb{R}^{n}\). Para ello, vamos a\(\vec{v}\) ser un vector de\(\mathbb{R}^{n}\), y necesitamos escribir\(\vec{v}\) como una combinación lineal\(\vec{u}_i\) de's Consideremos que la matriz\(A\) tiene los vectores\(\vec{u}_i\) como columnas:\[A = \left[ \begin{array}{rrr} \vec{u}_{1} & \cdots & \vec{u}_{n} \end{array} \right]\nonumber \]

    Por independencia lineal de los\(\vec{u}_i\)'s, la forma reducida de fila-escalón de\(A\) es la matriz de identidad. Por lo tanto, el sistema\(A\vec{x}= \vec{v}\) tiene una solución (única), por lo que\(\vec{v}\) es una combinación lineal de los\(\vec{u}_i\)'s.

    Para establecer el segundo reclamo, supongamos que\(m<n.\) Entonces dejando\(\vec{u}_{i_{1}},\cdots ,\vec{u}_{i_{k}}\) ser las columnas pivotantes de la matriz\[\left[ \begin{array}{ccc} \vec{u}_{1} & \cdots & \vec{u}_{m} \end{array} \right]\nonumber \] que sigue\(k\leq m<n\) y estas columnas\(k\) pivotantes serían una base para\(\mathbb{R}^{n}\) tener menos de\(n\) vectores, contrario a Corolario\(\PageIndex{3}\).

    Por último considerar la tercera pretensión. Si no\(\left\{ \vec{u}_{1},\cdots ,\vec{u}_{n}\right\}\) es linealmente independiente, entonces reemplace esta lista con\(\left\{ \vec{u}_{i_{1}},\cdots ,\vec{u}_{i_{k}}\right\}\) donde estas son las columnas pivotantes de la matriz\[\left[ \begin{array}{ccc} \vec{u}_{1} & \cdots & \vec{u}_{n} \end{array} \right]\nonumber \] Entonces\(\left\{ \vec{u}_{i_{1}},\cdots ,\vec{u}_{i_{k}}\right\}\) abarca\(\mathbb{R}^{n}\) y es linealmente independiente, por lo que es una base que tiene menos de\(n\) vectores otra vez contrario a Corolario\(\PageIndex{3}\) .

    El siguiente teorema se desprende de la afirmación anterior.

    Teorema\(\PageIndex{8}\): Existence of Basis

    Que\(V\) sea un subespacio de\(\mathbb{R}^n\). Entonces existe una base de\(V\) con\(\dim(V)\leq n\).

    Considerar Corolario\(\PageIndex{4}\) junto con Teorema\(\PageIndex{8}\). Vamos\(\dim(V) = r\). Supongamos que existe un conjunto independiente de vectores en\(V\). Si este conjunto contiene\(r\) vectores, entonces es una base para\(V\). Si contiene menos de\(r\) vectores, entonces los vectores se pueden agregar al conjunto para crear una base de\(V\). Del mismo modo, cualquier conjunto de expansión\(V\) que contenga más de\(r\) vectores puede tener vectores eliminados para crear una base de\(V\).

    Ilustramos este concepto en el siguiente ejemplo.

    Ejemplo\(\PageIndex{12}\): Extending an Independent Set

    Considera el conjunto\(U\) dado por\[U=\left\{ \left.\left[\begin{array}{c} a\\ b\\ c\\ d\end{array}\right] \in\mathbb{R}^4 ~\right|~ a-b=d-c \right\}\nonumber \] Entonces\(U\) es un subespacio de\(\mathbb{R}^4\) y\(\dim(U)=3\).

    Entonces\[S=\left\{ \left[\begin{array}{c} 1\\ 1\\ 1\\ 1\end{array}\right], \left[\begin{array}{c} 2\\ 3\\ 3\\ 2\end{array}\right] \right\},\nonumber \] es un subconjunto independiente de\(U\). Por lo tanto se\(S\) puede extender a una base de\(U\).

    Solución

    Para extender\(S\) a una base de\(U\), encontrar un vector en el\(U\) que no está en\(\mathrm{span}(S)\). \[\left[\begin{array}{rrr} 1 & 2 & ? \\ 1 & 3 & ? \\ 1 & 3 & ? \\ 1 & 2 & ? \end{array}\right]\nonumber \]

    \[\left[\begin{array}{rrr} 1 & 2 & 1 \\ 1 & 3 & 0 \\ 1 & 3 & -1 \\ 1 & 2 & 0 \end{array}\right] \rightarrow \left[\begin{array}{rrr} 1 & 0 & 0 \\ 0 & 1 & 0 \\ 0 & 0 & 1 \\ 0 & 0 & 0 \end{array}\right]\nonumber \]

    Por lo tanto,\(S\) puede extenderse a las siguientes bases de\(U\):\[\left\{ \left[\begin{array}{r} 1\\ 1\\ 1\\ 1\end{array}\right], \left[\begin{array}{r} 2\\ 3\\ 3\\ 2\end{array}\right], \left[\begin{array}{r} 1\\ 0\\ -1\\ 0\end{array}\right] \right\},\nonumber \]

    A continuación consideramos el caso de eliminar vectores de un conjunto de expansión para dar como resultado una base.

    Teorema\(\PageIndex{9}\): Finding a Basis from a Span

    Que\(W\) sea un subespacio. También supongamos eso\(W=span\left\{ \vec{w} _{1},\cdots ,\vec{w}_{m}\right\}\). Entonces existe un subconjunto del\(\left\{ \vec{w}_{1},\cdots ,\vec{w}_{m}\right\}\) cual es una base para\(W\).

    Prueba

    Let\(S\) denotar el conjunto de enteros positivos de tal manera que para\(k\in S,\) existe un subconjunto de\(\left\{ \vec{w}_{1},\cdots ,\vec{w}_{m}\right\}\) que consiste en exactamente\(k\) vectores que es un conjunto de expansión para\(W\). Así\(m\in S\). Elija el entero positivo más pequeño en\(S\). Llámenlo\(k\). Entonces existe\(\left\{ \vec{u}_{1},\cdots , \vec{u}_{k}\right\} \subseteq \left\{ \vec{w}_{1},\cdots ,\vec{w} _{m}\right\}\) tal que\(\text{span}\left\{ \vec{u}_{1},\cdots ,\vec{u} _{k}\right\} =W.\) si\[\sum_{i=1}^{k}c_{i}\vec{w}_{i}=\vec{0}\nonumber \] y no todos los\(c_{i}=0,\) entonces podrías escoger\(c_{j}\neq 0\), dividir por ello y resolver para\(\vec{u}_{j}\) en cuanto a los demás,\[\vec{w}_{j}=\sum_{i\neq j}\left( -\frac{c_{i}}{c_{j}}\right) \vec{w}_{i}\nonumber \] entonces podrías borrar\(\vec{w}_{j}\) de la lista y tener el mismo lapso. Cualquier combinación lineal involucrada\(\vec{w}_{j}\) sería igual a una en la que\(\vec{w}_{j}\) se reemplaza con la suma anterior, mostrando que podría haberse obtenido como una combinación lineal de\(\vec{w}_{i}\) for\(i\neq j\). Por lo tanto\(k-1\in S\) contrario a la elección de\(k\). De ahí que cada\(c_{i}=0\) y así\(\left\{ \vec{u}_{1},\cdots ,\vec{u} _{k}\right\}\) sea una base para\(W\) consistir en vectores de\(\left\{ \vec{w} _{1},\cdots ,\vec{w}_{m}\right\}\).

    El siguiente ejemplo ilustra cómo llevar a cabo este proceso de contracción que obtendrá un subconjunto de un lapso de vectores que es linealmente independiente.

    Ejemplo\(\PageIndex{13}\): Subset of a Span

    Dejado\(W\) ser el subespacio\[span\left\{ \left[ \begin{array}{r} 1 \\ 2 \\ -1 \\ 1 \end{array} \right] ,\left[ \begin{array}{r} 1 \\ 3 \\ -1 \\ 1 \end{array} \right] ,\left[ \begin{array}{r} 8 \\ 19 \\ -8 \\ 8 \end{array} \right] ,\left[ \begin{array}{r} -6 \\ -15 \\ 6 \\ -6 \end{array} \right] ,\left[ \begin{array}{r} 1 \\ 3 \\ 0 \\ 1 \end{array} \right] ,\left[ \begin{array}{r} 1 \\ 5 \\ 0 \\ 1 \end{array} \right] \right\}\nonumber \] Encuentra una base para la\(W\) cual consiste en un subconjunto de los vectores dados.

    Solución

    Puede usar la forma de escalón de fila reducida para lograr esta reducción. Formar la matriz que tiene los vectores dados como columnas. \[\left[ \begin{array}{rrrrrr} 1 & 1 & 8 & -6 & 1 & 1 \\ 2 & 3 & 19 & -15 & 3 & 5 \\ -1 & -1 & -8 & 6 & 0 & 0 \\ 1 & 1 & 8 & -6 & 1 & 1 \end{array} \right]\nonumber \]Luego toma la forma reducida de fila-escalón

    \[\left[ \begin{array}{rrrrrr} 1 & 0 & 5 & -3 & 0 & -2 \\ 0 & 1 & 3 & -3 & 0 & 2 \\ 0 & 0 & 0 & 0 & 1 & 1 \\ 0 & 0 & 0 & 0 & 0 & 0 \end{array} \right]\nonumber \]De ello se deduce que una base para\(W\)

    \[\left\{ \left[ \begin{array}{r} 1 \\ 2 \\ -1 \\ 1 \end{array} \right] ,\left[ \begin{array}{r} 1 \\ 3 \\ -1 \\ 1 \end{array} \right] ,\left[ \begin{array}{c} 1 \\ 3 \\ 0 \\ 1 \end{array} \right] \right\}\nonumber \]Dado que la primera, segunda y quinta columnas son obviamente una base para el espacio de columna de la, lo mismo es cierto para la matriz que tiene los vectores dados como columnas.

    Consideremos los siguientes teoremas respecto a un subespacio contenido en otro subespacio.

    Teorema\(\PageIndex{10}\): Subset of a Subspace

    Dejar\(V\) y\(W\) ser subespacios de\(\mathbb{R}^n\), y supongamos que\(W\subseteq V\). Entonces\(\dim(W) \leq \dim(V)\) con igualdad cuando\(W=V\).

    Teorema\(\PageIndex{11}\): Extending a Basis

    Dejar\(W\) ser cualquier subespacio distinto de cero\(\mathbb{R}^{n}\) y dejar\(W\subseteq V\) donde también\(V\) es un subespacio de\(\mathbb{R}^{n}\). Entonces cada base de se\(W\) puede extender a una base para\(V\).

    El comprobante se deja como ejercicio pero procede de la siguiente manera. Comience con una base para\(W,\left\{ \vec{w}_{1},\cdots ,\vec{w}_{s}\right\}\) y agregue vectores desde\(V\) hasta que obtenga una base para\(V\). No es que el proceso se detenga porque la dimensión de no\(V\) es más que\(n\).

    Considera el siguiente ejemplo.

    Ejemplo\(\PageIndex{14}\): Extending a Basis

    Dejar\(V=\mathbb{R}^{4}\) y dejar\[W=\mathrm{span}\left\{ \left[ \begin{array}{c} 1 \\ 0 \\ 1 \\ 1 \end{array} \right] ,\left[ \begin{array}{c} 0 \\ 1 \\ 0 \\ 1 \end{array} \right] \right\}\nonumber \] Extender esta base de\(W\) a una base de\(\mathbb{R}^{n}\).

    Solución

    Una manera fácil de hacer esto es tomar la forma de escalón de fila reducida de la matriz

    \[\left[ \begin{array}{cccccc} 1 & 0 & 1 & 0 & 0 & 0 \\ 0 & 1 & 0 & 1 & 0 & 0 \\ 1 & 0 & 0 & 0 & 1 & 0 \\ 1 & 1 & 0 & 0 & 0 & 1 \end{array} \right] \label{basiseq1}\]

    Observe cómo los vectores dados se colocaron como las dos primeras columnas y luego se extendió la matriz de tal manera que queda claro que el lapso de las columnas de esta matriz arroja todos\(\mathbb{R}^{4}\). Ahora determina las columnas pivotes. La forma reducida de fila-escalón es

    \[\left[ \begin{array}{rrrrrr} 1 & 0 & 0 & 0 & 1 & 0 \\ 0 & 1 & 0 & 0 & -1 & 1 \\ 0 & 0 & 1 & 0 & -1 & 0 \\ 0 & 0 & 0 & 1 & 1 & -1 \end{array} \right] \label{basiseq2}\]

    Por lo tanto, las columnas pivotes son\[\left[ \begin{array}{c} 1 \\ 0 \\ 1 \\ 1 \end{array} \right] ,\left[ \begin{array}{c} 0 \\ 1 \\ 0 \\ 1 \end{array} \right] ,\left[ \begin{array}{c} 1 \\ 0 \\ 0 \\ 0 \end{array} \right] ,\left[ \begin{array}{c} 0 \\ 1 \\ 0 \\ 0 \end{array} \right]\nonumber \]

    y ahora esto es una extensión de la base dada para\(W\) a una base para\(\mathbb{R}^{4}\).

    ¿Por qué funciona esto? Las columnas de\(\eqref{basiseq1}\) obviamente abarcan\(\mathbb{R }^{4}\). De hecho, el lapso de los cuatro primeros es el mismo que el lapso de los seis.

    Considera otro ejemplo.

    Ejemplo\(\PageIndex{15}\): Extending a Basis

    Dejar\(W\) ser el lapso de\(\left[ \begin{array}{c} 1 \\ 0 \\ 1 \\ 0 \end{array} \right]\) en\(\mathbb{R}^{4}\). Dejar\(V\) consistir en el lapso de los vectores\[\left[ \begin{array}{c} 1 \\ 0 \\ 1 \\ 0 \end{array} \right] ,\left[ \begin{array}{c} 0 \\ 1 \\ 1 \\ 1 \end{array} \right] ,\left[ \begin{array}{r} 7 \\ -6 \\ 1 \\ -6 \end{array} \right] ,\left[ \begin{array}{r} -5 \\ 7 \\ 2 \\ 7 \end{array} \right] ,\left[ \begin{array}{c} 0 \\ 0 \\ 0 \\ 1 \end{array} \right]\nonumber \] Encontrar una base para la\(V\) cual se extiende la base para\(W\).

    Solución

    Obsérvese que los vectores anteriores no son linealmente independientes, sino su envergadura, denotada como\(V\) es un subespacio que sí incluye el subespacio\(W\).

    Usando el proceso descrito en el ejemplo anterior, formar la siguiente matriz

    \[\left[ \begin{array}{rrrrr} 1 & 0 & 7 & -5 & 0 \\ 0 & 1 & -6 & 7 & 0 \\ 1 & 1 & 1 & 2 & 0 \\ 0 & 1 & -6 & 7 & 1 \end{array} \right]\nonumber \]

    A continuación encuentra su forma reducida de fila-escalón\[\left[ \begin{array}{rrrrr} 1 & 0 & 7 & -5 & 0 \\ 0 & 1 & -6 & 7 & 0 \\ 0 & 0 & 0 & 0 & 1 \\ 0 & 0 & 0 & 0 & 0 \end{array} \right]\nonumber \]

    De ello se deduce que una base para\(V\) consiste en los dos primeros vectores y el último. \[\left\{ \left[ \begin{array}{c} 1 \\ 0 \\ 1 \\ 0 \end{array} \right] ,\left[ \begin{array}{c} 0 \\ 1 \\ 1 \\ 1 \end{array} \right] ,\left[ \begin{array}{c} 0 \\ 0 \\ 0 \\ 1 \end{array} \right] \right\}\nonumber \]Así\(V\) es de dimensión 3 y tiene una base que extiende la base para\(W\).

    Espacio de fila, espacio de columnas y espacio nulo de una matriz

    Comenzamos esta sección con una nueva definición.

    Definición\(\PageIndex{9}\): Row and Column Space

    \(A\)Déjese ser una\(m\times n\) matriz. El espacio de columna de\(A\), escrito\(\mathrm{col}(A)\), es el lapso de las columnas. El espacio de filas de\(A\), escrito\(\mathrm{row}(A)\), es el lapso de las filas.

    Usando la forma de escalón de fila reducida, podemos obtener una descripción eficiente del espacio de fila y columna de una matriz. Considera el siguiente lema.

    Lema\(\PageIndex{1}\): Effect of Row Operations on Row Space

    Dejar\(A\) y\(B\) ser\(m\times n\) matrices tales que\(A\) puedan ser llevadas a\(B\) por\(\left[ \mbox{column} \right]\) operaciones elementales de fila. Entonces\(\mathrm{row}(A)=\mathrm{row}(B)\)\(\left[\mathrm{col}(A)=\mathrm{col}(B) \right]\).

    Prueba

    Demostraremos que lo anterior es cierto para las operaciones de fila, que se pueden aplicar fácilmente a las operaciones de columna.

    Dejar\(\vec{r}_1, \vec{r}_2, \ldots, \vec{r}_m\) denotar las filas de\(A\).

    • Si\(B\) se obtiene a partir de\(A\) un intercambiando dos filas de\(A\), entonces\(A\) y\(B\) tienen exactamente las mismas filas, entonces\(\mathrm{row}(B)=\mathrm{row}(A)\).
    • Supongamos\(p\neq 0\), y supongamos que para algunos\(j\)\(1\leq j\leq m\),,\(B\) se obtiene de\(A\) multiplicando fila\(j\) por\(p\). Entonces\[\mathrm{row}(B)=\mathrm{span}\{ \vec{r}_1, \ldots, p\vec{r}_{j}, \ldots, \vec{r}_m\}.\nonumber \] Desde que\[\{ \vec{r}_1, \ldots, p\vec{r}_{j}, \ldots, \vec{r}_m\} \subseteq\mathrm{row}(A),\nonumber \] se deduce eso\(\mathrm{row}(B)\subseteq\mathrm{row}(A)\). Por el contrario, ya\[\{ \vec{r}_1, \ldots, \vec{r}_m\}\subseteq\mathrm{row}(B),\nonumber \] que de ello se desprende\(\mathrm{row}(A)\subseteq\mathrm{row}(B)\). Por lo tanto,\(\mathrm{row}(B)=\mathrm{row}(A)\).

    Supongamos\(p\neq 0\), y supongamos que para algunos\(i\) y\(j\)\(1\leq i,j\leq m\),,\(B\) se obtiene\(A\) de sumando fila de\(p\) tiempo\(j\) a fila\(i\). Sin pérdida de generalidad, podemos asumir\(i<j\).

    Entonces\[\mathrm{row}(B)=\mathrm{span}\{ \vec{r}_1, \ldots, \vec{r}_{i-1}, \vec{r}_i+p\vec{r}_j, \ldots,\vec{r}_j,\ldots, \vec{r}_m\}.\nonumber \]

    Ya que de\[\{ \vec{r}_1, \ldots, \vec{r}_{i-1}, \vec{r}_i+p\vec{r}_{j}, \ldots, \vec{r}_m\} \subseteq\mathrm{row}(A),\nonumber \] ello se deduce que\(\mathrm{row}(B)\subseteq\mathrm{row}(A)\).

    Por el contrario, ya\[\{ \vec{r}_1, \ldots, \vec{r}_m\}\subseteq\mathrm{row}(B),\nonumber \] que de ello se desprende\(\mathrm{row}(A)\subseteq\mathrm{row}(B)\). Por lo tanto,\(\mathrm{row}(B)=\mathrm{row}(A)\).

    Considera el siguiente lema.

    Lema\(\PageIndex{2}\): Row Space of a reduced row-echelon form Matrix

    Dejar\(A\) ser una\(m \times n\) matriz y dejar\(R\) ser su forma reducida fila-escalón. Entonces las filas distintas de cero\(R\) forman una base de\(\mathrm{row}(R)\), y consecuentemente de\(\mathrm{row}(A)\).

    Este lema sugiere que podemos examinar la forma reducida fila-escalón de una matriz para obtener el espacio de filas. Consideremos ahora el espacio de columna. El espacio de columna se puede obtener simplemente diciendo que es igual al lapso de todas las columnas. Sin embargo, a menudo puede obtener el espacio de columna como la extensión de menos columnas que esta. Una variación del lema anterior proporciona una solución. Supongamos que\(A\) se reduce la fila a su forma reducida de fila-escalón\(R\). Identificar las columnas pivotes de\(R\) (columnas que tienen las principales), y tomar las columnas correspondientes de\(A\). Resulta que esto forma una base de\(\mathrm{col}(A)\).

    Antes de proceder a un ejemplo de este concepto, revisamos la definición de rango.

    Definición\(\PageIndex{10}\): Rank of a Matrix

    Anteriormente, definimos como\(\mathrm{rank}(A)\) el número de entradas principales en la forma fila-escalón de\(A\). Usando una comprensión de la dimensión y el espacio de filas, ahora podemos definir el rango de la siguiente manera:\[\mbox{rank}(A) = \dim(\mathrm{row}(A))\nonumber \]

    Considera el siguiente ejemplo.

    Ejemplo\(\PageIndex{16}\): Rank, Column and Row Space

    Encuentre el rango de la siguiente matriz y describa los espacios de columna y fila. \[A = \left[ \begin{array}{rrrrr} 1 & 2 & 1 & 3 & 2 \\ 1 & 3 & 6 & 0 & 2 \\ 3 & 7 & 8 & 6 & 6 \end{array} \right]\nonumber \]

    Solución

    La forma de fila-escalón reducida de\(A\) es\[\left[ \begin{array}{rrrrr} 1 & 0 & -9 & 9 & 2 \\ 0 & 1 & 5 & -3 & 0 \\ 0 & 0 & 0 & 0 & 0 \end{array} \right]\nonumber \] Por lo tanto, el rango es\(2\).

    Observe que las dos primeras columnas de\(R\) son columnas pivotables. Por la discusión que sigue a Lema\(\PageIndex{2}\), encontramos las columnas correspondientes de\(A\), en este caso las dos primeras columnas. Por lo tanto, una base para\(\mathrm{col}(A)\) es dada por\[\left\{\left[ \begin{array}{r} 1 \\ 1 \\ 3 \end{array} \right] , \left[ \begin{array}{r} 2 \\ 3 \\ 7 \end{array} \right] \right\}\nonumber \]

    Por ejemplo, considere la tercera columna de la matriz original. Se puede escribir como una combinación lineal de las dos primeras columnas de la matriz original de la siguiente manera. \[\left[ \begin{array}{r} 1 \\ 6 \\ 8 \end{array} \right] =-9\left[ \begin{array}{r} 1 \\ 1 \\ 3 \end{array} \right] +5\left[ \begin{array}{r} 2 \\ 3 \\ 7 \end{array} \right]\nonumber \]

    ¿Qué pasa con una descripción eficiente del espacio de filas? Por Lemma\(\PageIndex{2}\) sabemos que las filas distintas de cero de\(R\) crean una base de\(\mathrm{row}(A)\). Para la matriz anterior, el espacio de filas es igual\[\mathrm{row}(A) = \mathrm{span} \left\{ \left[ \begin{array}{rrrrr} 1 & 0 & -9 & 9 & 2 \end{array} \right], \left[ \begin{array}{rrrrr} 0 & 1 & 5 & -3 & 0 \end{array} \right] \right\}\nonumber \]

    Observe que el espacio de columna de\(A\) se da como el tramo de columnas de la matriz original, mientras que el espacio de filas de\(A\) es el tramo de filas de la forma de escalón de fila reducida de\(A\).

    Considera otro ejemplo.

    Ejemplo\(\PageIndex{17}\): Rank, Column and Row Space

    Encuentre el rango de la siguiente matriz y describa los espacios de columna y fila. \[\left[ \begin{array}{rrrrrr} 1 & 2 & 1 & 3 & 2 \\ 1 & 3 & 6 & 0 & 2 \\ 1 & 2 & 1 & 3 & 2 \\ 1 & 3 & 2 & 4 & 0 \end{array} \right]\nonumber \]

    Solución

    La forma reducida de fila-escalón es\[\left[ \begin{array}{rrrrrr} 1 & 0 & 0 & 0 & \frac{13}{2} \\ 0 & 1 & 0 & 2 & -\frac{5}{2} \\ 0 & 0 & 1 & -1 & \frac{1}{2} \\ 0 & 0 & 0 & 0 & 0 \end{array} \right]\nonumber \] y así lo es el rango\(3\). El espacio de fila viene dado por\[\mathrm{row}(A) = \mathrm{span} \left\{ \left[ \begin{array}{ccccc} 1 & 0 & 0 & 0 & \frac{13}{2} \end{array} \right], \left[ \begin{array}{rrrrr} 0 & 1 & 0 & 2 & -\frac{5}{2} \end{array} \right] , \left[ \begin{array}{rrrrr} 0 & 0 & 1 & -1 & \frac{1}{2} \end{array} \right] \right\}\nonumber \]

    Observe que las tres primeras columnas de la forma fila-escalón reducido son columnas pivotes. El espacio de columna es el tramo de las tres primeras columnas de la matriz original,\[\mathrm{col}(A) = \mathrm{span} \left\{ \left[ \begin{array}{r} 1 \\ 1 \\ 1 \\ 1 \end{array} \right], \; \left[ \begin{array}{r} 2 \\ 3 \\ 2 \\ 3 \end{array} \right] , \; \left[ \begin{array}{r} 1 \\ 6 \\ 1 \\ 2 \end{array} \right] \right\}\nonumber \]

    Considere la solución dada anteriormente para Ejemplo\(\PageIndex{17}\), donde el rango de\(A\) iguales\(3\). Observe que el espacio de fila y el espacio de columna tenían cada uno una dimensión igual a\(3\). Resulta que esto no es una coincidencia, y este resultado esencial es referido como el Teorema de Rango y se da ahora. Recordemos que definimos\(\mathrm{rank}(A) = \mathrm{dim}(\mathrm{row}(A))\).

    Teorema\(\PageIndex{12}\): Rank Theorem

    \(A\)Déjese ser una\(m \times n\) matriz. Entonces\(\mathrm{dim}(\mathrm{col} (A))\), la dimensión del espacio de columna, es igual a la dimensión del espacio de filas,\(\mathrm{dim}(\mathrm{row}(A))\).

    Todas las siguientes declaraciones siguen del Teorema de Rango.

    Corolario\(\PageIndex{5}\): Results of the Rank Theorem

    \(A\)Déjese ser una matriz. Entonces son ciertas las siguientes:

    1. \(\mathrm{rank}(A) = \mathrm{rank}(A^T)\).
    2. Para\(A\) de tamaño\(m \times n\),\(\mathrm{rank}(A) \leq m\) y\(\mathrm{rank}(A) \leq n\).
    3. Para\(A\) de tamaño\(n \times n\),\(A\) es invertible si y solo si\(\mathrm{rank}(A) = n\).
    4. Para matrices invertibles\(B\) y\(C\) de tamaño apropiado,\(\mathrm{rank}(A) = \mathrm{rank}(BA) = \mathrm{rank}(AC)\).

    Considera el siguiente ejemplo.

    Ejemplo\(\PageIndex{18}\): Rank of the Transpose

    Vamos\[A = \left[ \begin{array}{rr} 1 & 2 \\ -1 & 1 \end{array} \right]\nonumber \] Find\(\mathrm{rank}(A)\) y\(\mathrm{rank}(A^T)\).

    Solución

    Para encontrar primero la fila\(\mathrm{rank}(A)\) reducimos para encontrar la forma de fila-escalón reducida. \[A = \left[ \begin{array}{rr} 1 & 2 \\ -1 & 1 \end{array} \right] \rightarrow \cdots \rightarrow \left[ \begin{array}{rr} 1 & 0 \\ 0 & 1 \end{array}\right]\nonumber \]

    Por lo tanto el rango de\(A\) es\(2\). Ahora considere\(A^T\) dado por\[A^T = \left[ \begin{array}{rr} 1 & -1 \\ 2 & 1 \end{array} \right]\nonumber \] De nuevo remaremos reducir para encontrar la forma de fila-escalón reducida.

    \[\left[ \begin{array}{rr} 1 & -1 \\ 2 & 1 \end{array} \right] \rightarrow \cdots \rightarrow \left[ \begin{array}{rr} 1 & 0 \\ 0 & 1 \end{array} \right]\nonumber \]

    Se puede ver eso\(\mathrm{rank}(A^T) = 2\), lo mismo que\(\mathrm{rank}(A)\).

    Ahora definimos lo que se entiende por el espacio nulo de una\(m\times n\) matriz general.

    Definición\(\PageIndex{11}\): Null Space, or Kernel, of \(A\)

    El espacio nulo de una matriz\(A\), también denominado núcleo de\(A\), se define de la siguiente manera. \[\mathrm{null} \left( A\right) =\left\{ \vec{x} :A \vec{x} =\vec{0}\right\}\nonumber \]

    También se puede referir usando la notación\(\ker \left( A\right)\). De igual manera, podemos discutir la imagen de\(A\), denotada por\(\mathrm{im}\left( A\right)\). La imagen de\(A\) consiste en los vectores de los\(\mathbb{R}^{m}\) cuales “se golpean” por\(A\). La definición formal es la siguiente.

    Definición\(\PageIndex{12}\): Image of \(A\)

    La imagen de\(A\), escrita\(\mathrm{im}\left( A\right)\) es dada por\[\mathrm{im}\left( A \right) = \left\{ A\vec{x} : \vec{x} \in \mathbb{R}^n \right\}\nonumber \]

    Considerar\(A\) como un mapeo de\(\mathbb{R}^{n}\) a\(\mathbb{R}^{m}\) cuya acción viene dada por la multiplicación. El siguiente diagrama muestra este escenario. \[\overset{\mathrm{null} \left( A\right) }{\mathbb{R}^{n}}\ \overset{A}{\rightarrow }\ \overset{ \mathrm{im}\left( A\right) }{\mathbb{R}^{m}}\nonumber \]Como se indica,\(\mathrm{im}\left( A\right)\) es un subconjunto de\(\mathbb{R}^{m}\) while\(\mathrm{null} \left( A\right)\) es un subconjunto de\(\mathbb{R}^{n}\).

    Resulta que el espacio nulo y la imagen de\(A\) son ambos subespacios. Considera el siguiente ejemplo.

    Ejemplo\(\PageIndex{19}\): Null Space

    \(A\)Déjese ser una\(m\times n\) matriz. Entonces el espacio nulo de\(A\),\(\mathrm{null}(A)\) es un subespacio de\(\mathbb{R}^n\).

    Solución
    • Ya que\(A\vec{0}_n=\vec{0}_m\),\(\vec{0}_n\in\mathrm{null}(A)\).
    • Vamos\(\vec{x},\vec{y}\in\mathrm{null}(A)\). Entonces\(A\vec{x}=\vec{0}_m\) y\(A\vec{y}=\vec{0}_m\), así\[A(\vec{x}+\vec{y})=A\vec{x}+A\vec{y} = \vec{0}_m+\vec{0}_m=\vec{0}_m,\nonumber \] y así\(\vec{x}+\vec{y}\in\mathrm{null}(A)\).
    • Dejar\(\vec{x}\in\mathrm{null}(A)\) y\(k\in\mathbb{R}\). Entonces\(A\vec{x}=\vec{0}_m\), así\[A(k\vec{x}) = k(A\vec{x})=k\vec{0}_m=\vec{0}_m,\nonumber \] y así\(k\vec{x}\in\mathrm{null}(A)\).

    Por lo tanto por la prueba subespacial,\(\mathrm{null}(A)\) es un subespacio de\(\mathbb{R}^n\).

    La prueba de que\(\mathrm{im}(A)\) es un subespacio de\(\mathbb{R}^m\) es similar y se deja como ejercicio al lector.

    Ahora deseamos encontrar una manera de describir\(\mathrm{null}(A)\) para una matriz\(A\). Sin embargo, ¡encontrar no\(\mathrm{null} \left( A\right)\) es nuevo! Solo se está utilizando alguna terminología nueva, como\(\mathrm{null} \left( A\right)\) es simplemente la solución al sistema\(A\vec{x}=\vec{0}\).

    Teorema\(\PageIndex{13}\): Basis of null(A)

    Que\(A\) sea una\(m \times n\) matriz tal que\(\mathrm{rank}(A) = r\). Entonces el sistema\(A\vec{x}=\vec{0}_m\) cuenta con soluciones\(n-r\) básicas, proporcionando una base de\(\mathrm{null}(A)\) con\(\dim(\mathrm{null}(A))=n-r\).

    Considera el siguiente ejemplo.

    Ejemplo\(\PageIndex{20}\): Null Space of \(A\)

    Vamos\[A=\left[ \begin{array}{rrr} 1 & 2 & 1 \\ 0 & -1 & 1 \\ 2 & 3 & 3 \end{array} \right]\nonumber \] Find\(\mathrm{null} \left( A\right)\) y\(\mathrm{im}\left( A\right)\).

    Solución

    Para encontrar\(\mathrm{null} \left( A\right)\), simplemente necesitamos resolver la ecuación\(A\vec{x}=\vec{0}\). Este es el procedimiento habitual de escribir la matriz aumentada, encontrar la forma fila-escalón reducida y luego la solución. La matriz aumentada y la forma de escalón de fila reducida correspondiente son\[\left[ \begin{array}{rrr|r} 1 & 2 & 1 & 0 \\ 0 & -1 & 1 & 0 \\ 2 & 3 & 3 & 0 \end{array} \right] \rightarrow \cdots \rightarrow \left[ \begin{array}{rrr|r} 1 & 0 & 3 & 0 \\ 0 & 1 & -1 & 0 \\ 0 & 0 & 0 & 0 \end{array} \right]\nonumber \]

    La tercera columna no es una columna pivote, y por lo tanto la solución contendrá un parámetro. La solución al sistema\(A\vec{x}=\vec{0}\) viene dada por la\[\left[ \begin{array}{r} -3t \\ t \\ t \end{array} \right] :t\in \mathbb{R}\nonumber \] cual se puede escribir como\[t \left[ \begin{array}{r} -3 \\ 1 \\ 1 \end{array} \right] :t\in \mathbb{R}\nonumber \]

    Por lo tanto, el espacio nulo de\(A\) es todos múltiplos de este vector, que podemos escribir como\[\mathrm{null} (A) = \mathrm{span} \left\{ \left[ \begin{array}{r} -3 \\ 1 \\ 1 \end{array} \right] \right\}\nonumber \]

    Finalmente\(\mathrm{im}\left( A\right)\) es justo\(\left\{ A\vec{x} : \vec{x} \in \mathbb{R}^n \right\}\) y por lo tanto consiste en el lapso de todas las columnas de\(A\), es decir\(\mathrm{im}\left( A\right) = \mathrm{col} (A)\).

    Observe del cálculo anterior que las dos primeras columnas de la forma fila-escalón reducido son columnas pivotes. Así el espacio de columna es el lapso de las dos primeras columnas en la matriz original, y obtenemos\[\mathrm{im}\left( A\right) = \mathrm{col}(A) = \mathrm{span} \left\{ \left[ \begin{array}{r} 1 \\ 0 \\ 2 \end{array} \right], \; \left[ \begin{array}{r} 2 \\ -1 \\ 3 \end{array} \right] \right\}\nonumber \]

    Aquí hay un ejemplo más amplio, pero el método es completamente similar.

    Ejemplo\(\PageIndex{21}\): Null Space of \(A\)

    Vamos\[A=\left[ \begin{array}{rrrrr} 1 & 2 & 1 & 0 & 1 \\ 2 & -1 & 1 & 3 & 0 \\ 3 & 1 & 2 & 3 & 1 \\ 4 & -2 & 2 & 6 & 0 \end{array} \right]\nonumber \] Encuentra el espacio nulo de\(A\).

    Solución

    Para encontrar el espacio nulo, necesitamos resolver la ecuación\(AX=0\). La matriz aumentada y la correspondiente forma de fila reducida están dadas por

    \[\left[ \begin{array}{rrrrr|r} 1 & 2 & 1 & 0 & 1 & 0 \\ 2 & -1 & 1 & 3 & 0 & 0 \\ 3 & 1 & 2 & 3 & 1 & 0 \\ 4 & -2 & 2 & 6 & 0 & 0 \end{array} \right] \rightarrow \cdots \rightarrow \left[ \begin{array}{rrrrr|r} 1 & 0 & \frac{3}{5} & \frac{6}{5} & \frac{1}{5} & 0 \\ 0 & 1 & \frac{1}{5} & -\frac{3}{5} & \frac{2}{5} & 0 \\ 0 & 0 & 0 & 0 & 0 & 0 \\ 0 & 0 & 0 & 0 & 0 & 0 \end{array} \right]\nonumber \]De ello se deduce que las dos primeras columnas son columnas pivotes, y las tres siguientes corresponden a parámetros. Por lo tanto,\(\mathrm{null} \left( A\right)\) es dado por\[\left[ \begin{array}{c} \left( -\frac{3}{5}\right) s +\left( -\frac{6}{5}\right) t+\left( \frac{1}{5}\right) r \\ \left( -\frac{1}{5}\right) s +\left( \frac{3}{5}\right) t +\left( - \frac{2}{5}\right) r \\ s \\ t \\ r \end{array} \right] :s ,t ,r\in \mathbb{R}\text{.}\nonumber \] Escribimos esto en la forma\[s \left[ \begin{array}{r} -\frac{3}{5} \\ -\frac{1}{5} \\ 1 \\ 0 \\ 0 \end{array} \right] + t \left[ \begin{array}{r} -\frac{6}{5} \\ \frac{3}{5} \\ 0 \\ 1 \\ 0 \end{array} \right] + r \left[ \begin{array}{r} \frac{1}{5} \\ -\frac{2}{5} \\ 0 \\ 0 \\ 1 \end{array} \right] :s , t , r\in \mathbb{R}\text{.}\nonumber \] En otras palabras, el espacio nulo de esta matriz es igual al lapso de los tres vectores anteriores. Así\[\mathrm{null} \left( A\right) =\mathrm{span}\left\{ \left[ \begin{array}{r} -\frac{3}{5} \\ -\frac{1}{5} \\ 1 \\ 0 \\ 0 \end{array} \right] ,\left[ \begin{array}{r} -\frac{6}{5} \\ \frac{3}{5} \\ 0 \\ 1 \\ 0 \end{array} \right] ,\left[ \begin{array}{r} \frac{1}{5} \\ -\frac{2}{5} \\ 0 \\ 0 \\ 1 \end{array} \right] \right\}\nonumber \]

    Observe también que los tres vectores anteriores son linealmente independientes y así la dimensión de\(\mathrm{null} \left( A\right)\) es 3. Lo siguiente es cierto en general, el número de parámetros en la solución de\(AX=0\) es igual a la dimensión del espacio nulo. Recordemos también que el número de los principales en la forma de fila-escalón reducido es igual al número de columnas pivotantes, que es el rango de la matriz, que es el mismo que la dimensión de la columna o el espacio de fila.

    Antes de proceder a un teorema importante, primero definimos lo que se entiende por nulidad de una matriz.

    Definición\(\PageIndex{13}\): Nullity

    La dimensión del espacio nulo de una matriz se llama la nulidad, denotada\(\dim( \mathrm{null}\left(A\right))\).

    De nuestra observación anterior podemos ahora exponer un teorema importante.

    Teorema\(\PageIndex{14}\): Rank and Nullity

    \(A\)Déjese ser una\(m\times n\) matriz. Entonces\(\mathrm{rank}\left( A\right) + \dim( \mathrm{null}\left(A\right)) =n\).

    Ejemplo\(\PageIndex{22}\): Rank and Nullity

    Let\[A=\left[ \begin{array}{rrr} 1 & 2 & 1 \\ 0 & -1 & 1 \\ 2 & 3 & 3 \end{array} \right]\nonumber \]

    Encontrar\(\mathrm{rank}\left( A\right)\) y\(\dim( \mathrm{null}\left(A\right))\).

    Solución

    En el Ejemplo anterior\(\PageIndex{20}\) determinamos que la forma de fila-escalón reducida de\(A\) viene dada por\[\left[ \begin{array}{rrr} 1 & 0 & 3 \\ 0 & 1 & -1 \\ 0 & 0 & 0 \end{array} \right]\nonumber \]

    Por lo tanto el rango de\(A\) es\(2\). También determinamos que el espacio nulo de\(A\) está dado por\[\mathrm{null} (A) = \mathrm{span} \left\{ \left[ \begin{array}{r} -3 \\ 1 \\ 1 \end{array} \right] \right\}\nonumber \]

    Por lo tanto la nulidad de\(A\) es\(1\). Se deduce del teorema\(\PageIndex{14}\) que\(\mathrm{rank}\left( A\right) + \dim( \mathrm{null}\left(A\right)) = 2 + 1 = 3\), que es el número de columnas de\(A\).

    Concluimos esta sección con dos teoremas similares e importantes.

    Teorema\(\PageIndex{15}\)

    \(A\)Déjese ser una\(m\times n\) matriz. Los siguientes son equivalentes.

    1. \(\mathrm{rank}(A)=n\).
    2. \(\mathrm{row}(A)=\mathbb{R}^n\), es decir, las filas de\(A\) span\(\mathbb{R}^n\).
    3. Las columnas de\(A\) son independientes en\(\mathbb{R}^m\).
    4. La\(n\times n\) matriz\(A^TA\) es invertible.
    5. Existe una\(n\times m\) matriz\(C\) para que\(CA=I_n\).
    6. Si\(A\vec{x}=\vec{0}_m\) para algunos\(\vec{x}\in\mathbb{R}^n\), entonces\(\vec{x}=\vec{0}_n\).
    Teorema\(\PageIndex{16}\)

    \(A\)Déjese ser una\(m\times n\) matriz. Los siguientes son equivalentes.

    1. \(\mathrm{rank}(A)=m\).
    2. \(\mathrm{col}(A)=\mathbb{R}^m\), es decir, las columnas de\(A\) span\(\mathbb{R}^m\).
    3. Las filas de\(A\) son independientes en\(\mathbb{R}^n\).
    4. La\(m\times m\) matriz\(AA^T\) es invertible.
    5. Existe una\(n\times m\) matriz\(C\) para que\(AC=I_m\).
    6. El sistema\(A\vec{x}=\vec{b}\) es consistente para cada\(\vec{b}\in\mathbb{R}^m\).

    This page titled 4.10: Abarcando, Independencia Lineal y Base en R is shared under a CC BY 4.0 license and was authored, remixed, and/or curated by Ken Kuttler (Lyryx) via source content that was edited to the style and standards of the LibreTexts platform; a detailed edit history is available upon request.